Difference between revisions of "1975 AHSME Problems/Problem 16"

(Solution)
 
(6 intermediate revisions by the same user not shown)
Line 11: Line 11:
  
 
==Solution==
 
==Solution==
nothing yet :(
+
 
 +
Let's establish some ground rules...
 +
 
 +
 
 +
<math>a =</math> The first term in the geometric sequence.
 +
 
 +
 
 +
<math>r =</math> The ratio relating the terms of the geometric sequence.
 +
 
 +
 
 +
<math>n =</math> The nth value of the geometric sequence, starting at 1 and increasing as consecutive integer values.
 +
 
 +
 
 +
Using these terms, the sum can be written as:
 +
 
 +
<math>sum = a/(1-r) = 3</math>
 +
 
 +
 
 +
Let <math>x =</math> The positive integer that is in the reciprocal of the geometric ratio.
 +
 
 +
 
 +
This gives:
 +
 
 +
<math>3 = a/(1-(1/x))</math>
 +
 
 +
<math>3 = ax/(x-1)</math>
 +
 
 +
 
 +
Now through careful inspection we notice that when x = 3 the equation becomes
 +
 
 +
<math>3 =  a(3/2)</math>, where <math>a = 2</math>.
 +
 
 +
 
 +
Therefore <math>r = 1/x = 1/3</math>. Now we define the sum as <math>2 * (1/3)^{n-1}</math>.
 +
 
 +
 
 +
Now we simply add the <math>n = 1</math> and <math>n = 2</math> terms.
 +
 
 +
 
 +
<math>sum = 2(1/3)^{(1)-1} + 2(1/3)^{(2)-1} = 2 + 2/3 = 6/3 + 2/3 = 8/3</math>
 +
 
 +
 
 +
This gives <math>\boxed{\textbf{(C) } 8/3}</math>.
 +
 
 +
 
 +
~PhysicsDolphin
 +
 
 +
For more information on geometric sequences: https://artofproblemsolving.com/wiki/index.php/Geometric_sequence
  
 
==See Also==
 
==See Also==
 
{{AHSME box|year=1975|num-b=15|num-a=17}}
 
{{AHSME box|year=1975|num-b=15|num-a=17}}
 
{{MAA Notice}}
 
{{MAA Notice}}

Latest revision as of 07:27, 1 July 2024

Problem

If the first term of an infinite geometric series is a positive integer, the common ratio is the reciprocal of a positive integer, and the sum of the series is $3$, then the sum of the first two terms of the series is

$\textbf{(A)}\ \frac{1}{3} \qquad \textbf{(B)}\ \frac{2}{3} \qquad \textbf{(C)}\ \frac{8}{3} \qquad \textbf{(D)}\ 2           \qquad \textbf{(E)}\ \frac{9}{2} \qquad$

Solution

Let's establish some ground rules...


$a =$ The first term in the geometric sequence.


$r =$ The ratio relating the terms of the geometric sequence.


$n =$ The nth value of the geometric sequence, starting at 1 and increasing as consecutive integer values.


Using these terms, the sum can be written as:

$sum = a/(1-r) = 3$


Let $x =$ The positive integer that is in the reciprocal of the geometric ratio.


This gives:

$3 = a/(1-(1/x))$

$3 = ax/(x-1)$


Now through careful inspection we notice that when x = 3 the equation becomes

$3 =  a(3/2)$, where $a = 2$.


Therefore $r = 1/x = 1/3$. Now we define the sum as $2 * (1/3)^{n-1}$.


Now we simply add the $n = 1$ and $n = 2$ terms.


$sum = 2(1/3)^{(1)-1} + 2(1/3)^{(2)-1} = 2 + 2/3 = 6/3 + 2/3 = 8/3$


This gives $\boxed{\textbf{(C) } 8/3}$.


~PhysicsDolphin

For more information on geometric sequences: https://artofproblemsolving.com/wiki/index.php/Geometric_sequence

See Also

1975 AHSME (ProblemsAnswer KeyResources)
Preceded by
Problem 15
Followed by
Problem 17
1 2 3 4 5 6 7 8 9 10 11 12 13 14 15 16 17 18 19 20 21 22 23 24 25 26 27 28 29 30
All AHSME Problems and Solutions

The problems on this page are copyrighted by the Mathematical Association of America's American Mathematics Competitions. AMC logo.png